Intersection of any 2 intervals --> all intervals intersect

In summary, the author tries to solve a problem where some intervals in a family of intervals intersect. The author outlines a procedure for solving the problem where each interval is considered in turn and an induction step is used to solve for the intersection of all intervals in the family.
  • #1
Mr Davis 97
1,462
44

Homework Statement


Let ##\mathscr{F}## be a finite family of open or closed intervals in the line ##\mathbb{R}^1##. Show by an elementary proof that if any ##2## of them intersect, then all of them intersect.

Homework Equations

The Attempt at a Solution


Here is my attempt, where for now I'm just tackling the case for when all of the sets are open.

Since ##\mathscr{F}## is a finite family, let it have cardinality ##n##, and let's list the intervals in the family as ##F_1,F_2, \dots , F_n## such that, on the real line, the left bracket of ##F_1## is strictly to the left of the left bracket of ##F_2##, whose left bracket is strictly to the left of the left bracket of ##F_3##, and so on. Now, for the hypothesis suppose that ##F_1 \cap F_i \not = \emptyset## for ##i=2, \dots , n##. Then the right bracket of ##F_1## must be to the right of the left bracket of ##F_i## in order to intersect ##F_i##. This necessitates that the right bracket of ##F_1## be to the right of the left bracket of ##F_n##. Similarly, the right bracket of ##F_2## must be to the right of the left bracket of ##F_n##, and so on, and finally the right bracket of ##F_{n-1}## must be to the right of the left bracket of ##F_n##.

So, we see that every right bracket of ##F_i## where ##i=1,2, \dots, n-1## is to the right of the left bracket of ##F_n##. Now, the region between the left bracket of ##F_n## and the first right bracket to the right of the left bracket of ##F_n## is a region of intersection for all ##F_i## so that ##\bigcap F_i \not = \emptyset##
 
Last edited:
Physics news on Phys.org
  • #2
The proof statement must be missing some constraint, because a counterexample is the family ##(1,2),(3,5),(4,6)## for which the last two intersect but the first intersects neither of the other two.
 
  • #3
I assume
Mr Davis 97 said:
if any ##2## of them intersect
means: Let ##\mathscr{F}= \{\,I_k\,|\,1\leq k \leq n\,\}## with ##I_k\cap I_m \neq \emptyset ## for all ##k,m##.
 
  • Like
Likes Mr Davis 97
  • #4
fresh_42 said:
I assume

means: Let ##\mathscr{F}= \{\,I_k\,|\,1\leq k \leq n\,\}## with ##I_k\cap I_m \neq \emptyset ## for all ##k,m##.
That is the correct interpretation. Sorry if it was ambiguous
 
  • #5
Mr Davis 97 said:
That is the correct interpretation. Sorry if it was ambiguous
I think the terminus technicus for left and right bracket should be infimum and supremum.
 
  • Like
Likes Mr Davis 97
  • #6
fresh_42 said:
I think the terminus technicus for left and right bracket should be infimum and supremum.
Got it. I have a question with what I have. For simplicity in showing the main idea I just assumed that ##\inf F_1 < \cdots < \inf F_i < \cdots < \inf F_n##, where the inequalities are strict. The procedure I outlined involves moving sequentially from ##F_i## to ##F_{i+1}##. But what if, for example, ##\inf F_2 = \inf F_3##? This kind of ruins the procedure because I then don't really know whether to pick ##F_2## or ##F_3## first. This seems like a small issue, but how do I get around?
 
  • #7
I would proceed by induction. The cases ##n=1,2## are trivial. And in the induction step, you just have to add one other interval. Try to write it down for ##2 \to 3## and then it should work ##n \to n+1## as well. Maybe you could use an indirect proof for the induction step. I haven't read your proof though. The many left and rights and the wall of text made me dizzy. The idea to concentrate on open intervals was good, because if it works for the open versions, it will certainly work for the closed ones (I think). But you are right, you cannot assume ##\inf I_k < \inf I_m##, only ##\inf I_k \leq \inf I_m##. However, I would not order them. It makes things only seemingly easier.
 
  • #8
fresh_42 said:
I would proceed by induction. The cases ##n=1,2## are trivial. And in the induction step, you just have to add one other interval. Try to write it down for ##2 \to 3## and then it should work ##n \to n+1## as well. Maybe you could use an indirect proof for the induction step. I haven't read your proof though. The many left and rights and the wall of text made me dizzy. The idea to concentrate on open intervals was good, because if it works for the open versions, it will certainly work for the closed ones (I think). But you are right, you cannot assume ##\inf I_k < \inf I_m##, only ##\inf I_k \leq \inf I_m##. However, I would not order them. It makes things only seemingly easier.
Is the following easier to understand? Before I do anything with induction I want to see whether this proof holds up or not.

Since ##\mathscr{F}## is a finite family, let it have cardinality ##n##, and let's list the intervals in the family as ##F_1,F_2, \dots , F_n## such that ##\inf F_i < \inf F_{i+1}## (note that such an ordering can always be done). Now, from the hypothesis we can suppose that ##F_1 \cap F_i \not = \emptyset## for ##i=2, \dots , n##. Then ##\sup F_1 > \inf F_2## in order for ##F_1## to intersect ##F_2##. Since ##F_1## intersects all other intervals as well, we must have that ##\sup F_1 > \inf F_i##, and in particular, ##\sup F_1 > \inf F_n##. Similarly, ##\sup F_i > \inf F_n## for ##i=2,3,\dots , n-1##.

Now, the region between ##\inf F_n## and ##\min \{\sup F_i : i=1,2,\dots , n-1 \}## is a region of intersection for all ##F_i## so that ##\bigcap \mathscr{F} \not = \emptyset##
 
  • #9
I'm still skeptic. First, all your inequalities have to be ##\leq ## instead of ##<## and ##\geq ## instead of ##>##. Then it is not clear what happens at the interval broders: You decided to have open intervals, so infinma and suprema are not part of the interval, resp. what happens if they all are closed instead of open, e.g. if ##\sup F_1 = \inf F_2##. It might work, but it leaves a lot of open questions.

I would still proceed by induction. ##\cap_{k=1}^{n-1} F_k =: I_{n-1}## is again an interval, and the right border is the supremum of at least one ##F_{k_0}##, resp. the left border an infimum of at least one ##F_{k_0}##. Now you have only two intervals to discuss: ##F_{k_0}## and ##F_n## and you can consider all open and closed combinations, since there are only two of them.
 
Last edited:
  • #10
Here https://imgur.com/a/UlXhLRB there is a proof that I am trying to understand, before I do any kind of induction. My main question is, what exactly does the minmax and maxmin mean?
 
  • #11
Mr Davis 97 said:
Here https://imgur.com/a/UlXhLRB there is a proof that I am trying to understand, before I do any kind of induction. My main question is, what exactly does the minmax and maxmin mean?
5EfxMnN.png


For each pair ##(i,j) ## an element ## \alpha_{ij}\in J_i\cap J_j## is chosen. They exist be assumption. Now you can imagine them written as a symmetric matrix ##A :=(\alpha_{ij})_{1\leq i,j\leq n}##. Then ##\beta_1## is the highest number of all column minima and ##\beta_2## the smallest number among all column maxima. So first consider each column and determine the smallest and the highest entry of each. Among those determine the highest number among the collection of the smallest, and the smallest among the collection of the highest entries per column. It's possibly easier to see with an example of there or four such intervals.

Note that this proof doesn't operate with strict inequalities nor with open or closed intervals, only with the points in the middle of the intervals.
 

Attachments

  • 5EfxMnN.png
    5EfxMnN.png
    23.6 KB · Views: 517
Last edited:
  • #12
fresh_42 said:
View attachment 239289

For each pair ##(i,j) ## an element ## \alpha_{ij}\in J_i\cap J_j## is chosen. They exist be assumption. Now you can imagine them written as a symmetric matrix ##A :=(\alpha_{ij})_{1\leq i,j\leq n}##. Then ##\beta_1## is the highest number of all column minima and ##\beta_2## the smallest number among all column maxima. So first consider each column and determine the smallest and the highest entry of each. Among those determine the highest number among the collection of the smallest, and the smallest among the collection of the highest entries per column. It's possibly easier to see with an example of there or four such intervals.

Note that this proof doesn't operate with strict inequalities nor with open or closed intervals.
I see what you're saying when you write it out in words, but I don't see clearly how the notation works. What object exactly does ##\min a_{ij}## return? Is ##\max## acting on the ##\min a_{ij}## or is ##\max \min## taken as one object?
 
  • #13
I just constructed a matrix as described, it doesn't say much of the actual intervals, only that e.g. ##7 \in J_2\cap J_3## or ##0 \in J_1\cap J_5\,.## The diagonal is the intersection of an interval with itself. The fact that this intersection is not empty is equivalent that none of the intervals is empty. In my case we have ##k \in J_k\,.##
$$
\begin{bmatrix}-&1&4&6&2&0\\ -&4&2&7&8&5\\ -&6&7&3&1&1\\ -&2&8&1&4&9\\ -&0&5&1&9&5\\ \hline \\ min&0&2&1&1&0 \\ \hline \\ \beta_1&&2&&&\\ \hline \\ max&6&8&7&9&9 \\ \hline \\\beta_2&6&&&& \end{bmatrix}
$$
 
  • Like
Likes Mr Davis 97
  • #14
fresh_42 said:
I just constructed a matrix as described, it doesn't say much of the actual intervals, only that e.g. ##7 \in J_2\cap J_3## or ##0 \in J_1\cap J_5\,.## The diagonal is the intersection of an interval with itself. The fact that this intersection is not empty is equivalent that none of the intervals is empty. In my case we have ##k \in J_k\,.##
$$
\begin{bmatrix}-&1&4&6&2&0\\ -&4&2&7&8&5\\ -&6&7&3&1&1\\ -&2&8&1&4&9\\ -&0&5&1&9&5\\ \hline \\ min&0&2&1&1&0 \\ \hline \\ \beta_1&&2&&&\\ \hline \\ max&6&8&7&9&9 \\ \hline \\\beta_2&6&&&& \end{bmatrix}
$$
That helps a lot, but I am still a bit confused. Later in the proof, it says, for example, that ##z \le \max_{j=1, \dots, k} \alpha_{ij} \in J_i##. I thought that ##\max_{j=1, \dots, k} \alpha_{ij}## produced a list of the maximum elements in each column, so how could it be a single number?
 
  • #15
Mr Davis 97 said:
I thought that ##\max_{j=1, \dots, k} \alpha_{ij}## produced a list of the maximum elements in each column, so how could it be a single number?
It says for any (each) ##i\, : \,\forall \,i \, (z \leq \max_{j=1, \dots, k} \alpha_{ij} \in J_i)## so the condition involves only one certain index ##i##, however, any ##i##.
 
  • Like
Likes Mr Davis 97

What does it mean for all intervals to intersect?

When all intervals intersect, it means that there is at least one value that is contained within all of the intervals. In other words, there is a common overlap among all of the intervals.

Can any two intervals intersect?

Yes, any two intervals can intersect as long as they have at least one value in common. This can be visualized as the intervals overlapping on a number line.

What is the intersection of any two intervals?

The intersection of any two intervals is the set of all values that are contained within both intervals. This can be represented as the overlap of the two intervals on a number line.

How can you determine if all intervals intersect?

To determine if all intervals intersect, you can compare the intervals and see if there is any common overlap. If there is at least one value that is contained within all of the intervals, then they intersect.

Why is the intersection of any two intervals important?

The intersection of any two intervals is important because it allows us to identify common values or overlaps among different sets of data. This can be useful in various fields of study, such as statistics, mathematics, and computer science.

Similar threads

  • Calculus and Beyond Homework Help
Replies
4
Views
301
  • Calculus and Beyond Homework Help
Replies
2
Views
7K
  • Calculus and Beyond Homework Help
Replies
1
Views
499
  • Calculus and Beyond Homework Help
Replies
1
Views
1K
Replies
3
Views
1K
  • Calculus and Beyond Homework Help
Replies
5
Views
2K
  • Set Theory, Logic, Probability, Statistics
Replies
3
Views
877
  • Set Theory, Logic, Probability, Statistics
Replies
8
Views
1K
  • Calculus and Beyond Homework Help
Replies
1
Views
1K
  • Calculus and Beyond Homework Help
Replies
3
Views
2K
Back
Top